Below are the ratings for a sample of 12 hotels in small Ontario towns:

1.1 4.7 4.8 4.8 5.3 6.5 6.7 7.1 7.1 7.3 8.1 9.2

The critical values of this data set are given:

Q1=4.8 Q2=6.6 Q3=7.2

a) Determine the following and show all work: [3 marks]

i) interquartile range (IQR)

ii) upper boundary and lower boundary

iii) any outlier(s)


b) Create a modified box plot for the data, showing any outlier(s). [1 mark]


c) In order for a hotel to be recommended by travel blog TravelWell, its score must be the 80th percentile or higher. What is the minimum rating a hotel can achieve in order to be recommended by TravelWell based on this sample? [2 marks]

Answers

Answer 1

Answer:

a)

i) Interquartile range is   2.4  

ii) Upper boundary is 10.8  

The Lower boundary is 1.2  

iii) Outliers are points above the upper boundary or below the lower boundary  

There is one outlier: 1.1

b) The modified boxplot with one outlier is given in the diagram.                                                          

c) The 80th percentile is    

The minimum rating the hotel can get is 7.3

Step-by-step explanation:

a)

i) Interquartile range is  

IQR = Q3-Q1 = 7.2-4.8 = 2.4  

ii) Upper boundary = Q3+ 1.5 *IQR = 7.2+1.5*2.4 = 10.8  

Lower boundary = Q1 - 1.5*IQR = 4.8- 1.5*2.4 = 1.2  

iii) Outliers are points above the upper boundary or below the lower boundary  

There is one outlier: 1.1  

c) The 80th percentile is given by  

n= 12  

k = 0.80  

Index is given by k*n =0.80*12 = 9.6  

Round to nearest whole number, that is 10th  

80th percentile is the 10th number (arranged in ascending order)  

80th percentile = 7.3  

The minimum rating the hotel can get is 7.3.

b) The modified boxplot with one outlier is given as follows,  


Related Questions

a)out of 300 students In a class 60% of the students took physics and 35 students took chemistry and 20% of the students did not take any of this subject. how many students take both the subject​

Answers

Answer:

25 students take both subjects.

Step-by-step explanation:

Solve for 60% of 300 students:

60/100 = x/300

Cross multiply:

60 × 300 = 100 × x

18000 = 100x

Divide both sides by 100:

180 = x

Solve for 20% of 300 students:

20/100 = x/300

20 × 300 = 100 × x

6000 = 100x

60 = x

Solve for the percentage of students in chemistry:

x/100 = 35/300

x × 300 = 100 × 35

300x = 3500

x = 11.66666...7

x = about 11.7%

Find the difference in percentages:

100 - 60 - 20 - 11.7

8.3

8.3% take both subjects

Solve for 8.3% of students:

8.3/100 = x/300

8.3 × 300 = 100 × x

2490 = 100x

24.9

About 25 students

Check your work by adding all the students together (to get to 300):

25 + 60 + 180 + 35

300 students total

This is correct!

This trapezium is drawn on a centimetre grid.
Find the area of the trapezium.

Answers

Answer:

A = 18 units²

Step-by-step explanation:

The area (A) of a trapezium is calculated as

A = [tex]\frac{1}{2}[/tex] h ( b₁ + b₂ )

where h is the perpendicular height and b₁, b₂ the parallel bases

By counting squares

h = 3, b₁ = 3, b₂ = 9 , then

A = [tex]\frac{1}{2}[/tex] × 3 × (3 + 9) = 1.5 × 12 = 18 units²

in a survey 50% like cold drinks 30% like hot drinks and 40% like juice. Likewise 20% of them like cold and hot drinks 18% like cold drinks and juice, 12% like hot drinks and juice and 5% like all three drinks. represent the above information in a venn diagram and find the percent of people who like:

i) at least one of the drinks
ii) exactly two types of drinks
iii) exactly one type of drink
iv) none of the drinks

Answers

Answer:

Step-by-step explanation:

C -> Percentage of people who take cold drinks

H -> Percentage of people who take hot drinks

J - > Percentage of people who take Juice

i) At least one of drinks = 17 + 3 +15 +18 +7 + 15 + 5 = 75%

ii)  Exactly two types of drinks = Percentage of people who take cold  and hot drinks + Percentage of people who take cold and Juice + Percentage of people who take hot drinks and Juices

= 15 + 13 + 7

= 35%

iii) Exactly one type of drink =  Percentage of people who take only cold drinks + Percentage of people who take only hot drinks + Percentage of people who take only juices

= 17 + 3 + 15

= 35%

iv) None of the drinks = Total percentage - at least one of the drinks

                                    = 100 - 75

                                    = 25%

What is the midpoint of the segment shown below?
10
O A. (-3,-11)
O B. (-9,-)
10
(-12, -3)
O C. (-9.-11)
(3.-8)
OD. (-:-)

Answers

O C. (-9.-11) is THE ANSWER

the dual bar chart below shows how a group of 190 students travel to school. given that 50 students, said they travel to school by car complete the table below

Answers

The image of the bar chart is missing and so i have attached it.

Answer:

Bus = 75

Cycle = 20

Walk = 45

Step-by-step explanation:

We are told that 50 students said they travel to school by car.

However, in the bar chart, we see that we have a portion for boys and girls

Under car, when we count along the y-axis, we will see that, there are 6 units for boys and 4 units for girls.

Thus, to find the value of a unit since number of people that took cars is 50, we have;

(6x + 4x)/2 = 50

Where x is the value of one unit. Thus;

10x/2 = 50

5x = 50

x = 50/5

x = 10

Thus, a unit is 10

Under bus;

Boys have 7 units while girls have 8 units.

Thus;

Frequency by bus = ((7 × 10) + (8 × 10))/2 = 150/2 = 75

Under cycle;

Boys have 1 unit while girls have 3 units.

Thus;

Frequency by cycle = ((1 × 10) + (3 × 10))/2 = 20

Under walk;

Boys have 5 units while girls have 4 units.

Thus;

Frequency by cycle = ((5 × 10) + (4 × 10))/2 = 45

The ratio of flour to water is 2 to 1 for biscuits Is Lisa has 4 cups of flour how much water should she add to make biscuits

Answers

Answer:

she should add 2 cups of water.

Step-by-step explanation:

2 : 1 = 4 : 2

Lisa should add 2 cups of water

what is the cosine of 122.2

Answers

cosine 122.2 = -0.532876276......

Use the parabola tool to graph the quadratic function f(x)=−x2+4. Graph the parabola by first plotting its vertex and then plotting a second point on the parabola.

Answers

Answer:

see below

Step-by-step explanation:

f(x) = -x^2 +4

The vertex form is

y = a(x-h)^2 +k

Rewriting

f(x) = -(x-0)^2 +4

The vertex is (0,4) and a = -1

Since a is negative we know the parabola opens downward

f(x) = -(x^2 -4)

We can find the zeros

0 = -(x^2 -2^2)

This is the difference of squares

0 = -(x-2)(x+2)

Using the zero product property

x-2 =0   x+2 =0

x=2   x=-2

(2,0)  (-2,0) are the zeros of the parabola and 2 other points on the parabola

We have the maximum ( vertex) and the zeros and know that it opens downward, we can graph the parabola

Answer:

Your vertex is (4,0)

Step-by-step explanation:

Can someone help me with this math homework please!

Answers

Answer:

Second option

As the x-values go to negative infinity, the function´s values go to positive. infinity.

--------------------------

Hope it helps...

have a great day!!

Answer:

(B) As the x-values go to negative infinity, the function's values go to positive infinity.

Step-by-step explanation:

The x-values the answer choices are talking about are the values on the x-axis.

Looking at the graph, when the x-values go to negative infinity (meaning it keeps on going left, or negative), the function goes up, meaning the function goes to positive infinity.

When the x-values go to positive infinity (meaning it keeps on going right, or positive), the function goes up, meaning the function goes to positive infinity.

Out of all the answer choices, only B agrees with the observations written above.

Hope that helps (●'◡'●)

Find f(-3) for f(x) = 4(2)^x
O A. -32

O B. 1/2

O C. -24

O D. 1/8

Answers

Answer:

B. 1/2

General Formulas and Concepts:

Pre-Algebra

Order of Operations: BPEMDAS

Brackets Parenthesis Exponents Multiplication Division Addition Subtraction Left to Right

Algebra I

Functions

Function Notation

Step-by-step explanation:

Step 1: Define

Identify

f(x) = 4(2)ˣ

Step 2: Evaluate

Substitute in x [Function f(x)]:                                                                           f(-3) = 4(2)⁻³Exponents:                                                                                                        f(-3) = 4(1/8)Multiply:                                                                                                             f(-3) = 1/2

what is 3x10x178 I need help asap

Answers

Answer:

3x10=30   30x178=5,430

Step-by-step explanation:

QUESTION:- what is 3x10x178

ANSWER:-

[tex]3 \times 10 \times 178 \\ 30 \times 178 \\ 5340 \: \: answer[/tex]

Can someone help me with this math homework please!

Answers

Answer:

f(n+1) = f(n) - 5

Step-by-step explanation:

Just find some relationship between 2 numbers that are next to each other.

Which table of values could be generated by the equation 10x+5y=15? (Will give brainlest and 21 points)

Answers

Answer:

(For the image) A

Step-by-step explanation:

A car travels 600 km in 6 hours. at what rate of speed is the car traveling?

Answers

Step-by-step explanation:

100Km/hours......V/T

Find the length of side BC give your answer to three significant figures

Answers

Answer:

19.4 cm

Step-by-step explanation:

Hi there!

This is a right triangle. We're given an angle, the side adjacent to the angle and we're solving for the hypotenuse. Given this information, we can use the cosine ratio:

[tex]cos\theta=\frac{adj}{hyp}[/tex]

Plug in the given angle and side

[tex]cos71=\frac{6.3}{BC}\\BC=\frac{6.3}{cos71} \\BC=19.4[/tex]

Therefore, the length of BC is 19.4 cm when rounded to 3 significant figures.

I hope this helps!

What is the range of f(x)=4^x

Answers

Answer:

B

Step-by-step explanation:

At - infinity, the function will tend to 0 and at +infinity, the function will tend to +infinity. Those are the two extremas of the function and extremas define the range. Range is all positive real number


use a double angle or half angle identity to find the exact value of each expression

Answers

Answer:

Step-by-step explanation:

There are 2 very distinct and important things that we need to know before completing the problem. First is that we are given that the cos of an angle is 1/3 (adjacent/hypotenuse) and it is in the first quadrant. We also need to know that the identity for sin2θ = 2sinθcosθ.

We already know cos θ = 1/3, so we need now find the sin θ. The sin ratio is the side opposite the angle over the hypotenuse, and the side we are missing is the side opposite the angle (we do not need to know the angle; it's irrelevant). Set up a right triangle in the first quadrant and label the base with a 1 (because the base is the side adjacent to the angle), and the hypotenuse with a 3. Find the third side using Pythagorean's Theorem:

[tex]3^2=1^2+y^2[/tex] which simplifies to

[tex]9=1+y^2[/tex] and

[tex]y^2=8[/tex] so

[tex]y=\sqrt{8}=2\sqrt{2}[/tex] so that's the missing side. Now we can easily determine that

[tex]sin\theta=\frac{2\sqrt{2} }{3}[/tex]

Now we have everything we need to fill in the identity for sin2θ:

[tex]2sin\theta cos\theta=2(\frac{2\sqrt{2} }{3})(\frac{1}{3})[/tex] and multiply all of that together to get

[tex]2sin\theta cos\theta=\frac{4\sqrt{2} }{9}[/tex]

Grandma is making a quilt. She has 540 cm of fabric to border the quilt. What is the greatest possible area for the quilt?
Question 1 options:


11 664 cm^2


18225 cm^2


72900 cm^2


291600 cm^2

Show your work:

Answers

Answer:

18225 cm²

Step-by-step explanation:

Divide 540 by 4 to get the length of all sides

540/4 = 135

Square 135 to get the max possible size

135² = 18225

18225 cm²  is the greatest possible area for the quilt.

What is area?

The measurement that expresses the size of a region on a plane or curved surface is called area. Surface area refers to the area of an open surface or the boundary of a three-dimensional object, whereas the area of a plane region or plane area refers to the area of a form or planar lamina.

Given

Divide 540 by 4 to obtain the length of all sides

540/4 = 135

Square 135 to acquire the max possible size

135² = 18225

18225 cm²  is the greatest possible area for the quilt.        

To learn more about area refer to:

https://brainly.com/question/25292087

#SPJ2

Three red balls, 5 green balls and a number of blue balls are put together in a sac. One ball is picked at random from the sac. If the probability of picking a red ball is 1|6, find the a) The number of blue balls in sac. B) the probability of picking a green ball​

Answers

Answer:

total balls = 18 .... 3/x = 1/6

blue = 10 ... 18-(5+3) = 10

p of green = 5/18 = .277

Step-by-step explanation:

Find cosθ+cos3θ+cos5θ+cos7θ by using the Sum-to-Product Formula.
Please also show your work as well. Thanks!

Answers

Answer:

[tex] \rm\displaystyle 4\cos( \theta) \cos \left( {2\theta} \right) \cos \left( {4 \theta } \right) [/tex]

Step-by-step explanation:

I assume the question want us to rewrite cosθ+cos3θ+cos5θ+cos7θ by using Sum-to-Product Formula and note that it's not an equation therefore θ can never be specified

===========================

so we want to rewrite cosθ+cos3θ+cos5θ+cos7θ by using Sum-to-Product Formula the good news is that the number of the function of the given expression is even so there's a way to do so, rewrite the expression in parentheses notation:

[tex] \rm\displaystyle \left( \cos( \theta) + \cos(3 \theta) \right) + \left(\cos(5 \theta) + \cos(7 \theta) \right)[/tex]

recall that,Sum-to-Product Formula of cos function:

[tex] \rm \boxed{\displaystyle \cos( \alpha ) + \cos( \beta ) = 2 \cos \left( \frac{ \alpha + \beta }{2} \right) \cos \left( \frac{ \alpha - \beta }{2} \right) }[/tex]

notice that we have two pair of function with which we can apply the formula thus do so,

[tex] \rm\displaystyle \left( 2\cos \left( \frac{ \theta + 3 \theta}{2} \right)\cos \left( \frac{ \theta - 3 \theta}{2} \right) \right) + \left(2\cos \left( \frac{5 \theta + 7 \theta}{2} \right) \cos \left( \frac{5 \theta - 7 \theta}{2} \right) \right)[/tex]

simplify addition:

[tex] \rm\displaystyle \left( 2\cos \left( \frac{4 \theta}{2} \right)\cos \left( \frac{ - 2\theta }{2} \right) \right) + \left(2\cos \left( \frac{12 \theta }{2} \right) \cos \left( \frac{ - 2 \theta}{2} \right) \right)[/tex]

simplify division:

[tex] \rm\displaystyle \left( 2\cos \left( {2 \theta} \right)\cos \left( { - \theta } \right) \right) + \left(2\cos \left( {6 \theta } \right) \cos \left( { - \theta} \right) \right)[/tex]

By Opposite Angle Identities we acquire:

[tex] \rm\displaystyle \left( 2\cos \left( {2 \theta} \right)\cos \left( { \theta } \right) \right) + \left(2\cos \left( {6 \theta } \right) \cos \left( { \theta} \right) \right)[/tex]

factor out 2cosθ:

[tex] \rm\displaystyle 2 \cos( \theta) (\cos \left( {2 \theta} \right) + \cos \left( {6 \theta } \right) )[/tex]

once again apply Sum-to-Product Formula which yields:

[tex] \rm\displaystyle 2 \cos( \theta) (2\cos \left( {4\theta} \right) \cos \left( {2 \theta } \right) )[/tex]

distribute:

[tex] \rm\displaystyle 4\cos( \theta) \cos \left( {2\theta} \right) \cos \left( {4 \theta } \right) [/tex]

and we're done!

Find cos 0
A. 15/8
B. 15/17
C. 8/15
D. 8/17

Answers

Answer:

A.15/8

Step-by-step explanation:

the answer is 15/8

Answer:

D.

[tex]{ \tt{ \cos( \theta) = \frac{adjacent}{hypotenuse} }} \\ \\ { \tt{ \cos( \theta) = \frac{8}{ \sqrt{ {15}^{2} + {8}^{2} } } }} \\ \\ { \tt{ \cos( \theta) = \frac{8}{ \sqrt{289} } }} \\ \\ { \tt{ \cos( \theta) = \frac{8}{17} }}[/tex]

If f is continuous for all x, which of the following integrals necessarily have the same value?

Answers

Answer:

B

Step-by-step explanation:

Given the three integrals, we want to determine which integrals necessarily have the same value.

We can let the first integral be itself.

For the second integral, we can perform a u-substitution. Let u = x + a. Then:

[tex]\displaystyle du = dx[/tex]

Changing our limits of integration:

[tex]u_1=(0)+a=a \text{ and } u_2 = (b+a)+a = b+2a[/tex]

Thus, the second integral becomes:

[tex]\displaystyle \int_{0}^{b+a}f(x+a)\, dx = \int_a^{b+2a} f(u)\, du[/tex]

For the third integral, we can also perform a u-substitution. Let u = x + c. Then:

[tex]\displaystyle du = dx[/tex]

And changing our limits of integration:

[tex]\displaystyle u_1=(a-c)+c=a \text{ and } u_2=(b-c)+c=b[/tex]

Thus, our third integral becomes:

[tex]\displaystyle \int_{a-c}^{b-c}f(x+c)\, dx = \int_{a}^{b} f(u)\, du[/tex]

Since the only difference between f(x) and f(u) is the variable and both the first and third integral have the same limits of integration, our answer is B.

If the speed of an object in motion is doubled, its kinetic energy becomes how many times the original kinetic energy

Answers

Answer: Becomes four times

Step-by-step explanation:

Given

Speed is doubled for a moving object

Suppose initial speed is u

Increased speed is 2u

Kinetic Energy is given by

[tex]\Rightarrow K=0.5mu^2[/tex]

When speed is doubled

[tex]\Rightarrow K'=0.5m(2u)^2\\\Rightarrow K'=(0.5mu^2)\times 4\\\Rightarrow K'=4K[/tex]

Kinetic energy becomes four times

State what additional information is required in order to know that the triangle in the image below are congruent for the reason given…

Reason: HL Postulate

Answers

Answer:

FG ≈ FL (Both are hypotenuse, supposed to be equal in order to the congruency to become HL)

Answered by GAUTHMATH

A house on the market was valued at $472,000. After several years, the value increased by 19%. By how much did the house's value increase in dollars? What is the current value of the house?

Answers

Step-by-step explanation:

Increase in dollars

19/100 x 472.000 = $89,670

and the current value house is $472,000 + $89,670 = $561,680

the tens digit of a two digit number is 5 greater the units digit. If you subtract double the reversed number from it, the result is a fourth of the original number. Find the original number.

Answers

Given:

The tens digit of a two digit number is 5 greater the units digit.

If you subtract double the reversed number from it, the result is a fourth of the original number.

To find:

The original number.

Solution:

Let n be the two digit number and x be the unit digit. Then tens digit is (x+5) and the original number is:

[tex]n=(x+5)\times 10+x\times 1[/tex]

[tex]n=10x+50+x[/tex]

[tex]n=11x+50[/tex]

Reversed number is:

[tex]x\times 10+(x+5)\times 1=10x+x+5[/tex]

[tex]x\times 10+(x+5)\times 1=11x+5[/tex]

If you subtract double the reversed number from it, the result is a fourth of the original number.

[tex]11x+50-2(11x+5)=\dfrac{1}{4}(11x+50)[/tex]

[tex]11x+50-22x-10=\dfrac{1}{4}(11x+50)[/tex]

[tex]40-11x=\dfrac{1}{4}(11x+50)[/tex]

Multiply both sides by 4.

[tex]160-44x=11x+50[/tex]

[tex]160-50=11x+44x[/tex]

[tex]110=55x[/tex]

Divide both sides by 55.

[tex]\dfrac{110}{55}=x[/tex]

[tex]2=x[/tex]

The unit digit is 2. So, the tens digit is [tex]2+5=7[/tex].

Therefore, the original number is 72.

Given: x - 7 > -2.

Choose the solution set.

A. {x | x R, x > 14}
B. {x | x R, x > -5}
C. {x | x R, x > 5}
D. {x | x R, x > -9}

Answers

SoLuTiOn~

Given set:- x - 7 > - 2

Solving It:-

x - 7 > - 2

x > -2 + 7 [Here '7' is greater than '-2' So Sign Changes To Positive]

x > 5

So Correct Solution Set Will Be

Option C= {x | x R, x > 5}

Hope This Helps You

When AG = 16 ft, find the area of the region that is NOT shaded. Round to the nearest tenth.​

Answers

Answer:

730.88

Step-by-step explanation:

Area of the entire circle = pi * r^2

r = 16

Area = 3.14 * 16^2

Area = 803.84

1/4 of the circle contains the shaded area. It's area = 1/4 * 803.84

Area of 1/4 circle =

200.96

the area of the triangle

Area = 1/2 AG * G?

AG and G? are equal

Area = 1/2 * 16^2

Area = 128

Area of 1/4 circle - area of the triangle = area of the shaded portion

shaded portion = 200.95 - 128

Shaded Portion = 72.96

So the area of the unshaded part

unshaded = 803.84 - 72.96

Unshaded = 730.88

need some help with this​

Answers

Answer: The answer to that problem is the last equation (y = 4x - 7).

Reasoning: Since the slope is 4, it goes with “x”. The y-intercept is -7 and it goes to the end of the equation.

Answer:

y=4x-7

Step-by-step explanation:

here,

the equation of straight line in slope intercept form is;

y=mx+c

( m= slope

c= y-intercept )

soo..

the question has asked for slope 4 i.e. m=4

and y- intercept -7 i.e. c= -7

now.

the required equation is

y= 4x-7

mark me brainliest and follow me ... please

2/5(1/3x-15/8)-1/3(1/2-2/3x)

Answers

Answer:

[tex]\frac{16}{45}x-\frac{11}{12}[/tex]

Step-by-step explanation:

We are given that an expression

[tex]\frac{2}{5}(1/3x-15/8)-\frac{1}{3}(1/2-2/3x)[/tex]

We have to find the equivalent expression.

[tex]\frac{2}{5}(\frac{1}{3}x-\frac{15}{8})-\frac{1}{3}(\frac{1}{2}-\frac{2}{3}x)[/tex]

[tex]\frac{2}{5}\times \frac{1}{3}x-\frac{2}{5}\times \frac{15}{8}-\frac{1}{3}\times \frac{1}{2}-\frac{1}{3}\times (-\frac{2}{3}x)[/tex]

Using the the property

[tex]a\cdot (c-b)=a\cdot c-a\cdot b[/tex]

[tex]\frac{2}{5}(1/3x-15/8)-\frac{1}{3}(1/2-2/3x)[/tex]

[tex]=\frac{2}{15}x-\frac{3}{4}-\frac{1}{6}+\frac{2}{9}x[/tex]

[tex]=\frac{6x+10x}{45}+\frac{-9-2}{12}[/tex]

[tex]=\frac{16}{45}x-\frac{11}{12}[/tex]

[tex]\frac{2}{5}(1/3x-15/8)-\frac{1}{3}(1/2-2/3x)=\frac{16}{45}x-\frac{11}{12}[/tex]

Other Questions
which photo synthetic process can occur at night?A.the reaction at photosystem 1B.the formation of NADHC.the calvin cycle D.the reaction at photosystem 2 When a Select Case statement executes, the value of the test expression is compared with the values that follow each of the _______ keywords. John is a typical student. He usually on the lessons.A.pays attentionB.concentratesC.does D.understands Can someone please help What did Elizabeth Fry do ? 15) Three primary activities of a program are: A) Variables, Operators, and Key Words B) Lines, Statements, and Punctuation C) Input, Processing, and Output D) Integer, Floating-point and Character E) None of the above Write a method rearrange that takes a queue of integers as a parameter and rearranges the order of the values so that all of the even values appear before the odd values and that otherwise preserves the original order of the list. For example, suppose a queue called q stores this sequence of values: front [3, 5, 4, 17, 6, 83, 1, 84, 16, 37] back Then the call of rearrange(q); should rearrange the queue to store the following sequence of values: front [4, 6, 84, 16, 3, 5, 17, 83, 1, 37] back Notice that all of the evens appear at the front of the queue followed by the odds and that the order of the evens is the same as in the original list and the order of the odds is the same as in the original list. You may use one stack as auxiliary storage. I need two examples of rounding to the thousandths place. SHOW ALL WORK! Which of the following is a correct statement regarding the standard unmodified opinion audit report? Group of answer choices The auditor's responsibility paragraph includes a statement that the auditors are responsible for selecting the appropriate accounting principles. The format of the audit report for public and nonpublic entities are identical. The audit report includes the name of the lead partner on the audit. The auditor's responsibilities paragraph includes a statement that the auditor considers internal controls when designing the audit procedures performed. A tile manufacturer has supplied the following data: Boxes of tiles produced and sold 520,000 Sales revenue $ 2,132,000 Variable manufacturing expense $ 650,000 Fixed manufacturing expense $ 464,000 Variable selling and administrative expense $ 260,000 Fixed selling and administrative expense $ 312,000 Net operating income $ 446,000 What is the company's unit contribution margin Which situation best illustrates the economic concept of opportunity cost?O A. A business spends money on new computers, so it can't affordnew office furniture.B. A business invests in finding new customers rather than rewardingcurrent customers.C. A business plans to invest all of its profits back into the companyto keep growing.D. A business looks for workers overseas, so it has to hire amultilingual staff. If the cost, C, for manufacturing x units of a certain product is given by c=x^2-5x+65, find the number of units manufactured at a cost of 13,865. Suppose beak size currently ranges from 6-10mm. If directional selection favors larger beaks (up to a max of 15mm), which statement is true? Polynomial: 3x^4 + 5x - 4; Divisor: x - 1 1. Match the words with their correct meanings. a. communicate i. a person who spends most of the time with you b. tantrums ii. worthy of attention, striking, notably unusual c. companion iii. share or exchange information with others d. accomplish iv. a difficult or unlucky situation e. inspire V. a great victory or achievement f. remarkable vi. continuing to try g. determination vii. persuade, encourage, motivate tie h. triumph viii. childish display of bad temper i. adversity ix. very large e j. overwhelming x. will power, resolution k. persistence xi. achieve or complete successfully. On her summer abroad in France, Jane bought a pair of shoes for 54.82 euros. The store owner only had francs to give her as change. She gave him 55 euros. How much did he give her back in francs (I GOT AN 80% )1.(D) Ricardo has r minutes left on his cell-phone card. He wants to call his friend and still have 8 minutes left over to make calls later. Which expression describes the length of time he can talk to his friend? D) r 82.(D) Lorraine rode her bike one-sixth the distance Jeremy rode his bike. If b represents the distance Jeremy rode, which expression represents the distance Lorraine rode her bike? D) b --- 63.(A) Raoul rode an elevator up the floors. He entered on the 5'thand exited on the 34'th sloor. Which equation shows an equality between two different ways of expressing the Raoul exited ? A) 34 = 5 + e4.(B) Hank went to an apple orchard and paid an entrance fee of $5 and bought a pounds of apples at $2 per pound. Hank paid a total of $19.Which equation shows an equality between two different ways of expressing how much Hank paid in all? B) 5 + 2a = 195.(A) Chens house is 53 years old. This is 5 more than twice the age of Sunnys house, h.Which equation shows an equality between two different ways of expressing the age of Chens house? A) 53 = 2h + 56.(A) Mandy and four of her friends want to see a movie. If m represents the cost for one person to see a movie, which expression represents the cost for Mandy and her friends to see a movie? A) 5m7.(WRONG ANSWER IS D) Natalie and Juan collect marbles. The number of marbles that Natalie has is 4 more than 3 times the number of marbles Juan has. If m represents the number of marbles Juan has, which of the following expressions represents the number of marbles Natalie has?A) 4m + 3B) 3m + 4C) 3m 4D) DO NOT PICK ITS WRONG 8.(WRONG ANSWER IS D) Hannah baked twice the number of cookies Sam baked. Sam baked 36 cookies. Let c represent the number of cookies Hannah baked.Which equation shows an equality between two different ways of expressing how many cookies Sam baked?A) 36 = c 2B) 36 = c + 2C) 36 = c 2D)DO NOT PICK ITS WRONG9.(B) There were c members in the chess club before 8 more people joined. Which expression describes the number of members in the chess club now? B) c + 810.(C)C) P --- + 1.5 4 (all the correct answers are next to the numbers and it says do not pick which everyone's where wrong which was 7 and 8 i hope this helps you) what is 2 1/2 divided by 1/3 {pls hurry the teacher is not letting us use brainly} -0.6the decimal in equivalent fraction form I will literally give you my life if you. help me with these integer questions-